If you're seeing this message, it means we're having trouble loading external resources on our website.

If you're behind a web filter, please make sure that the domains *.kastatic.org and *.kasandbox.org are unblocked.

Main content

Course: LSAT > Unit 1

Lesson 5: Analytical Reasoning – Worked examples

Ordering setup | New info-must be true | Worked example

Watch a demonstration of one way to approach a question that asks you to identify a condition that would force an element into a specific position on an ordering setup on the Analytical Reasoning section of the LSAT.

Want to join the conversation?

  • blobby green style avatar for user Allie
    This can be quickly solved by understanding that L can only be in 602 or 603. Therefore the choice that puts any other monument into 603 would force L into 602.
    (12 votes)
    Default Khan Academy avatar avatar for user
  • blobby green style avatar for user nyplacnew
    I think this question can be quickly solved, just like the previous question, by realizing S is the free-lancer of the team who is not restricted by any rules. So the added rule restricting S would most likely restrict everything.
    (7 votes)
    Default Khan Academy avatar avatar for user
  • blobby green style avatar for user AlliKat
    Can someone explain how this is not a trick question? The answer can also be B, because of the first rule. L cannot occur in 601 with G. If the rule said "Later" vs. "later year" than this would make sense. The wordings insinuate two different scenarios.
    (2 votes)
    Default Khan Academy avatar avatar for user
  • piceratops seedling style avatar for user thea
    For this question, technically, doesn't G have to be in 601 since L comes right after G in the rules?
    (2 votes)
    Default Khan Academy avatar avatar for user
  • blobby green style avatar for user michelemayrebaula87
    how about H was placed at 604 , it forces L to be on 2nd .

    G/M are partners because M cant be placed on the sixth , therefore S should be on the sixth .

    G L F H S
    M
    please enlighten me
    (2 votes)
    Default Khan Academy avatar avatar for user
  • old spice man green style avatar for user Katiefawnd
    what about on the GMS part
    (1 vote)
    Default Khan Academy avatar avatar for user
  • blobby green style avatar for user cbelden01
    Why can't the answer be B? If rule one is [G...L...F], and L is located in 602 isn't G automatically forced to be in 601 or else it violates the rule.
    (1 vote)
    Default Khan Academy avatar avatar for user
    • aqualine ultimate style avatar for user NewYorker
      The question states where L must be. If G is in 601, L could be in either 602 or 603, it doesn't force L to only be in 602. According to the rule, L just needs to come after G, so either of those spots works. Because the question is asking where L must be based on the information, it rules out choice B.
      Putting S in 603, forces L to be in the 602 spot because it cannot go anywhere else.
      Hopefully that helps clear things up a little.
      (1 vote)
  • blobby green style avatar for user sophie longo
    Is there a reason G/M/S is circled but F/H/S is not?
    (0 votes)
    Default Khan Academy avatar avatar for user

Video transcript

make sure that you've already watched the setup video in which we created this initial diagram using the rules and the deductions that we made from those rules L must be the monument that was begun in 602 if which one of the following is true this question is essentially asking us to find the choice that if it's true forces L to be in 602 since there could be many conditions that push L into the 602 spot it would make more sense to evaluate each choice one at a time and test them quickly with our pencil instead of trying to predict the answer the wrong choices won't force L into 602 and the answer will be a condition that leads to deductions one of which is that L must be in 602 all right a says that F was begun in 605 if this is true then we know that age would be in 604 but we wouldn't know anything else for sure for example G could be in 601 or 602 so L wouldn't have to be in 602 B if G was begun in 601 we already don't know whether G is paired with M or with s in 601 and Al could be in 2 or 3 and so we hit a dead end here choice C is that H was begun in 604 this doesn't tell us anything F could be in 603 or 605 for example G could be in 601 or 602 so this will not allow us to determine that L has to be in 602 d-m was begun in 601 this is similar to I be was wrong because we already don't know whether M is paired with G or with s in 601 which means that L could be 2 or 3 and finally he tells us that s was begun in 603 well if s is in 603 that means that 601 has to be GM as a pair and since F and H would have to be the two that are in four and five that forces L to be in 602 or you can think of it as there's nowhere else that L could go because if we tried to put L in 604 for example that would put F into 605 and then H would be dangling in limbo somewhere so e is the answer that forces L to be the monument that was begun in 602